Thomas' Calculus 13th Edition

Published by Pearson
ISBN 10: 0-32187-896-5
ISBN 13: 978-0-32187-896-0

Chapter 10: Infinite Sequences and Series - Section 10.8 - Taylor and Maclaurin Series - Exercises 10.8 - Page 618: 16

Answer

$(\dfrac{x}{2})-\dfrac{1}{2^3 \cdot 3!} x^3+\dfrac{1}{2^{5} \cdot 5!}x^5-...$

Work Step by Step

Consider the Maclaurin Series for $\sin x$ as follows: $ \sin (x)=\Sigma_{n=0}^\infty \dfrac{(-1)^n x^{2n+1}}{(2n+1)!}$ Plug $\dfrac{x}{2}$ instead of $x$ in the above series. This implies that $ \sin (\dfrac{x}{2})=\Sigma_{n=0}^\infty \dfrac{(-1)^n (\dfrac{x}{2})^{2n+1}}{(2n+1)!}$ and $ \sin (\dfrac{x}{2})=(\dfrac{x}{2})-\dfrac{1}{2^3 \cdot 3!} x^3+\dfrac{1}{2^{5} \cdot 5!}x^5-...$
Update this answer!

You can help us out by revising, improving and updating this answer.

Update this answer

After you claim an answer you’ll have 24 hours to send in a draft. An editor will review the submission and either publish your submission or provide feedback.